If L stays open, then any of the following could be true EXCEPT:

zia305 on June 24, 2020

Explanation

Can someone please explain the right answer?

Reply
Create a free account to read and take part in forum discussions.

Already have an account? log in

Skylar on June 24, 2020

@zia305, happy to help!

This question tells us that L is open.

Rule #4 tells us that L and R cannot both be open. Since we already know that L is open, this means that R must be closed.

Rule #1 tells us that R and N cannot both be closed. Since we already know that R is closed, this means that N must be open.

Therefore, (B) is correct.

Does that make sense? Hope it helps! Please let us know if you have any other questions!